Energy conservation in the classically forbidden region

In summary, the conversation discusses the concept of energy conservation in quantum mechanics, specifically in regards to a particle approaching a finite potential step or inside a potential well. The particle has a finite probability to penetrate into the classically forbidden region where the potential is greater than the energy. It is explained that the Lagrangian is time-translational invariant, allowing for energy to be conserved. The issue of detecting the particle inside the classically forbidden region is also addressed, with the explanation that the act of detection will effectively collapse the wave function into a new wave function localized within the region. It is also noted that energy cannot be attributed to a specific position in standard quantum mechanics, and a potential energy term is introduced in the non-standard Bohm
  • #1
QuasiParticle
74
1
Consider a particle approaching a finite potential step or inside a potential well. As we know, the particle has a finite probability to penetrate into the region where E < V. The probability remains finite arbitrarily deep into the classically forbidden region. Suppose we set a detector far from the step and wait for a very long time. When we finally detect a particle, what saves the law of conservation of energy?

Unless we have an absolutely crazy potential, the Lagrangian is time-translational invariant, so energy should be conserved. What am I not getting?

The issue was discussed, e.g., here:

physics.stackexchange.com/questions/11188/can-a-particle-be-physically-observed-inside-a-quantum-barrier

I do not think the explanation about the penetration depth is sufficient, since we can set the detector arbitrarily far from the step.
 
Last edited:
Physics news on Phys.org
  • #2
As you point out, E is conserved. In the classically forbidden region V > E, implying the kinetic energy p2/2m is negative, p is imaginary and the particle does not propagate, it's exponentially damped.

In order to be detected, the particle has to cause a transition in the detector. The detector also conserves energy, and so if the particle lacks sufficient energy to cause the transition, it will not be detected.
 
  • #3
First of all, there is no region where E<V, just as there is no region where E>V. That's because E is NOT a function of position x. Instead, E is a functional of the whole wave function at ALL points at once.

Second, if you decide to detect the particle inside the classically forbidden region, nobody can stop you to try to do that, and there is a finite probability that you will be successfull. But in the case of successfull detection, the act of detection will effectively collapse the wave function into a new wave function localized within the classically forbidden region only. The collapse will be such that energy after the collapse will be equal to energy before the collapse.
 
  • #4
Demystifier said:
First of all, there is no region where E<V, just as there is no region where E>V. That's because E is NOT a function of position x. Instead, E is a functional of the whole wave function at ALL points at once.
E is constant, but V is a function which depends on the position (and maybe some other variables). Just like there are regions where 2<f(x) with f(x)=x^2, there can be regions with E<V(x) and E>V(x).
 
  • #5
mfb said:
E is constant, but V is a function which depends on the position (and maybe some other variables). Just like there are regions where 2<f(x) with f(x)=x^2, there can be regions with E<V(x) and E>V(x).
Yes, you are right. What I really wanted to say is that, given E and psi(x), in standard QM E cannot be interpreted as energy at position x. That's because, in standard QM, a particle with energy E and wave function psi(x) does not have a well-defined position x.

Attributing energy to a position x makes sense in the non-standard Bohmian interpretation, but then (instead of introducing negative kinetic energy) one should think of energy as a sum of positive kinetic energy p^2/2m, classical potential energy V(x), and quantum potential energy Q(x).
 
  • #6
Very good, Demystifier, in your effort to avoid non-standard interpretations you have invented a "quantum potential" Q(x) which does not appear in textbooks. You have also decided that the position variable x should not be called position. Well call it anything you like. E is constant and does not depend on it anyway. I am mystified.
 
  • #7
The question seems related to one that I have had-- if I prepare a particle in a spin state "up", then I send it through a Stern-Gerlach instrument at 90 degrees to that spin, I have a 50% chance of deflecting the particle left or right, associated with a state of left or right spin instead of "up." Let's say it deflects so as to have "left" spin. Does that not imply that the apparatus itself has picked up the angular momentum change? Or is it so impossible to measure the angular momentum change in the apparatus that we need an interpretation to tell us what the discrepancy is in the first place? (Like, in the Bohm approach, could we have started with a particle in a superposition of left and right spin that is actually one or the other but we had no way to determine that from the details of its preparation, so if which way it goes is predetermined, then there might not have been any angular momentum discrepancy in the first place?)
 
  • #8
Bill_K said:
Very good, Demystifier, in your effort to avoid non-standard interpretations you have invented a "quantum potential" Q(x) which does not appear in textbooks.

He was actually addressing the non standard Bohmian QM when referring to that Q(x).
 
  • #9
Bill_K said:
Very good, Demystifier, in your effort to avoid non-standard interpretations you have invented a "quantum potential" Q(x) ...
Thanks, but it was invented by David Bohm, who was much more clever and famous than me.

Bill_K said:
... which does not appear in textbooks.
Remarkably, it appears in one of the most famous and most standard textbooks of physics - Feynman lectures on physics. See Eq. (21.38) in vol. 3 of that textbook.
 
  • #10
Bill_K said:
I am mystified.
That's good because, according to Bohr, those who are not mystified by quantum mechanics do not yet understand it.
 
  • Like
Likes Spinnor

What is the classically forbidden region?

The classically forbidden region is a region in quantum mechanics where a particle does not have enough energy to overcome a potential barrier. In classical mechanics, the particle would not be able to pass through this region, but in quantum mechanics, there is a small probability of the particle tunneling through the barrier.

Why is energy conservation important in the classically forbidden region?

Energy conservation is important in the classically forbidden region because it ensures that the total energy of the system remains constant. This means that even though a particle may tunnel through a barrier, the overall energy of the system remains the same.

How does energy conservation affect the probability of tunneling in the classically forbidden region?

Energy conservation does not directly affect the probability of tunneling in the classically forbidden region. However, it is a fundamental principle in quantum mechanics and must be taken into account when calculating the probability of tunneling.

Can energy be conserved in the classically forbidden region even if a particle tunnels through a barrier?

Yes, energy can still be conserved in the classically forbidden region even if a particle tunnels through a barrier. This is because the total energy of the system remains constant, even though the particle may have gained or lost energy during the tunneling process.

How does the potential barrier affect energy conservation in the classically forbidden region?

The potential barrier plays a crucial role in energy conservation in the classically forbidden region. It determines the amount of energy required for a particle to overcome the barrier and the probability of tunneling through it. The shape and height of the barrier can also affect the amount of energy the particle may gain or lose during tunneling.

Similar threads

  • Quantum Physics
Replies
12
Views
1K
  • Quantum Physics
Replies
9
Views
1K
Replies
4
Views
948
  • Quantum Physics
Replies
3
Views
1K
  • Quantum Physics
Replies
1
Views
3K
Replies
4
Views
5K
  • Quantum Physics
Replies
3
Views
2K
Replies
24
Views
9K
  • Quantum Physics
Replies
3
Views
1K
  • Quantum Physics
2
Replies
41
Views
5K
Back
Top